Está en la página 1de 12

Solucionario Cuarta Fase V ONEM

Comisión de Olimpiadas de la Sociedad Matemática Peruana

Huampanı́, Noviembre 2008

Los enunciados y soluciones de este documento fueron elaborados por: John Cuya, Israel
Dı́az, Claudio Espinoza, Jorge Tipe y Sergio Vera.
Este documento está disponible libremente en http://www.onemperu.wordpress.com

1. Nivel 1
Problema 1.- ¿Cuántos números abc de tres dı́gitos distintos cumplen la siguiente propiedad?

“ al reemplazar el dı́gito mayor por el dı́gito 1 se obtiene un múltiplo de 30 ”

(Jorge Tipe)

Solución:
Tenemos que el dı́gito mayor no puede ser c, dado que al hacer el intercambio, el número
ab1 no puede ser múltiplo de 30. Tenemos ahora que el mayor de los dı́gitos solo puede ser
a o b.
Si el dı́gito mayor es a, al hacer el intercambio, el número 1bc es múltiplo de 30, y puede
tomar los valores 120, 150 y 180. Si el número después del intercambio fuese 120, entonces
el número inicial seria a20, con a > 2 (dado que es el dı́gito mayor), tendrı́amos entonces
7 números de esta forma. Trabajando de la misma manera, no es dı́ficil obtener que hay
4 números con número intercambiado 150; y 1 número con número intercambiado 180. Es
decir, hay 12 números en este caso.
Si el dı́gito mayor es b, al hacer el intercambio, el número a1c es múltiplo de 30, y puede
tomar los valores 210, 510 y 810. Si el número después del intercambio fuese 510, entonces
el número incial seria 5b0, con b > 5 (dado que es el dı́gito mayor), tendrı́amos entonces
4 números de esta forma. Trabajando de la misma manera, no es dı́ficil obtener que hay
7 números con número intercambiado 210; y 1 número con número intercambiado 810. Es
decir, hay 12 números en este caso.
Por lo tanto, en total existen 12 + 12 = 24 números que cumplen las condiciones del
problema.

Problema 2.- Se escriben los números naturales desde el 1 hasta el 9 inclusive y luego se
pintan usando los colores rojo, azul y verde. Cada número se pinta con un solo color,
de tal modo que cada número pintado de rojo es igual a la suma de un número pintado

1
Comisión de Olimpiadas de la Sociedad Matemática Peruana 2

de azul más un número pintado de verde. ¿Cuál es la máxima cantidad de números que
se pueden pintar de rojo?

(Israel Dı́az)

Solución:
Sean a, b y c las cantidades de números pintados de rojo, azul y verde, respectivamente.
Claramente a + b + c = 9. Supongamos que sea posible que a ≥ 5, luego b + c ≤ 4. La
cantidad de números rojos es a lo más bc, pues b números azules y c números verdes generan
bc sumas (algunas de las cuales se pueden repetir), es por eso que a ≤ bc. Sin embargo
b+c≤4 =⇒ bc ≤ 4 =⇒ a ≤ 4,
lo cual es una contradicción pues supusimos al inicio que a ≥ 5. Por lo tanto se debe cumplir
lo contrario, es decir, la cantidad de puntos rojos es a lo más 4, veamos un ejemplo de que
esto es posible:
azules: 1,2,3
verdes: 4,6
rojos: 5,7,8,9
este ejemplo cumple pues 5 = 1 + 4, 7 = 1 + 6, 8 = 2 + 6 y 9 = 3 + 6. Por la tanto la máxima
cantidad de puntos rojos es 4.

Problema 3.- Decimos que un entero positivo m es fierito si existe un número entero pos-
itivo N tal que la suma de las cifras de N es m, y además N es divisible por m + 2008.
a) Halla un número fierito mayor que 1000.
b) Halla un número fierito menor que 100.

(Israel Dı́az)

Solución:
a) Hagamos que m + 2008 = 10000, con esto m = 7992, ahora podemos encontrar el
número N = 1111
| {z · · · 11} 0000, que evidentemente es múltiplo de 10000 y la suma de sus
7992 veces
cifras es m = 7992. Por lo tanto 7992 > 1000 es fierito.
b) Vamos a buscar un valor de m < 50. Consideremos un número de la forma
N = abcdabcd · · · abcd,
en donde el bloque de cifras abcd aparece k veces. Es fácil ver que N es divisible por
abcd. Para que N sea múltiplo de m + 2008, vamos a hacer que
abcd = m + 2008 (1)
Comisión de Olimpiadas de la Sociedad Matemática Peruana 3

ahora, solo tendremos que conseguir que la suma de cifras de N sea m, luego

k(a + b + c + d) = m (2)
Como m < 50, podemos notar que a = 2 y b = 0, luego las ecuaciones (1) y (2) quedan
reducidas a

cd = m + 8
k(2 + c + d) = m

Si k = 1 tenemos que c + d + 10 = cd, luego 10 = 9c, que no es posible pues c es una


cifra. No hay soluciones en este caso.
Si k = 2 tenemos que 2c + 2d + 12 = cd, luego d + 12 = 8c de donde d = 4 y c = 2;
además m = 24 − 8 = 16.
Considerando el número N = 202142024, deducimos que 16 es fierito, pues 20242024 es
múltiplo de 16 + 2008 = 2024, y la suma de sus cifras es 16.

Problema 4.- Andrés y Victor juegan en un tablero de 7 × 7, escribiendo por turnos 0 ó 1


en alguna casilla desocupada. Andrés inicia el juego. Andrés gana el juego si logra que
aparezcan seis números iguales en fila, en columna o en diagonal. Además, Victor tiene
la opción de no jugar su turno si ası́ lo desea. Demuestra que Victor tiene una manera
de jugar de tal modo que Andrés no le puede ganar.

(John Cuya)

Solución:
Enumeramos las columnas del 1 al 7, de izquierda a derecha y las filas del 1 al 7, de arriba
hacia abajo. Luego, etiquetamos las casillas del tablero de la siguiente manera

1 2 3 4 5 6 7

P A A X 1
M N B 4 B Y Z 2
1 2 C C 5 6 7 3
D 3 5 D 4
1 2 3 E E 6 7 5
X Y F 4 F N P 6
Z G G M 7
La estrategia de Victor será, cada vez que Andrés coloque un número 0 ó 1 en una de las
casillas etiquetadas, en el siguiente turno Victor colocará un número distinto en la otra casilla
etiquetada con el mismo número o la misma letra. De este modo Andrés nunca podrá ga-
nar ya que para colocar 6 números iguales consecutivas en la columna 1, las casillas con
los números 1 deben tener los mismos números, pero la estrategia de Victor hace que esto
nunca ocurra, lo mismo en las otras columnas y filas. Sólo hay 6 diagonales posibles donde
Comisión de Olimpiadas de la Sociedad Matemática Peruana 4

se pueden colocar 6 números consecutivos iguales, las cuales también estan cubiertas con las
casillas M, N, P, X, Y y Z, en la estrategia de Victor. Las casillas sin etiquetar funcionan
como un solo grupo, que en total tiene 9 casillas (cantidad impar), entonces en este caso
cada vez que Andrés coloque un número en este grupo, Victor va a pasar, ası́ la estrategia
funciona correctamente y Andrés nunca podrá ganar.
Comisión de Olimpiadas de la Sociedad Matemática Peruana 5

2. Nivel 2
Problema 1.- Un profesor de matemáticas escribe el número 1 en la pizarra y le dice a su
alumno Gomito:
“ Puedes cambiar el número escrito en la pizarra por el número que resulta al multipli-
carlo por 2 o por 3 y sumarle 1, y puedes hacer esta operación cuantas veces quieras”.
Haciendo solamente estos cambios, sucesivamente,
a) ¿Es posible que Gomito obtenga el número 2008?
b) ¿Es posible que Gomito obtenga el número 2009?

(John Cuya)

Solución:

a) Sı́ es posible, y podemos hallar la secuencia de pasos procediendo de manera inversa:

2008 ← 669 ← 334 ← 111 ← 55 ← 27 ← 13 ← 4 ← 1

b) No es posible. Supongamos que sea posible y sea n el número que borró Gomito antes
de escribir 2009. Luego 2n+1 = 2009 o 3n+1 = 2009, pero la segunda ecuación no tiene
soluciones enteras, por lo tanto 2n + 1 = 2009, y por ende n = 1004. Sea m el número
que borró Gomito antes de escribir 1004, entonces 2m + 1 = 1004 o 3m + 1 = 1004, pero
ninguna de las ecuaciones anteriores tiene solución entera, esto es una contradicción
pues Gomito siempre escribe números enteros.

Problema 2.- Iván marca algunos puntos de una recta de tal modo que se cumple la si-
guiente propiedad:
“Siempre que Iván escoge tres puntos marcados, hay dos de ellos cuya distancia es
menor que 3 y hay dos de ellos cuya distancia es mayor que 3”.
¿Cuál es la mayor cantidad de puntos que puede marcar Iván?

(Jorge Tipe)

Solución:
Vamos a demostrar que Iván puede marcar como máximo 4 puntos, y un ejemplo serı́a
marcando los siguientes cuatro puntos:
2 2 2
A B C D

este ejemplo cumple, pues escogiendo tres puntos marcados cualesquiera hay dos de ellos
que se diferencian en 2, y dos que se diferencian en 4.
Ahora, probaremos que Iván no puede marcar 5 o más puntos. Basta probar que con 5
puntos no es posible.
Comisión de Olimpiadas de la Sociedad Matemática Peruana 6

Supongamos que los puntos A, B, C, D, E están ubicados en la recta, y en ese mismo or-
den. Analicemos los tres primeros puntos A, B, C. Como alguna de las distancias AB, BC y
AC es mayor que 3, y la mayor de esas distancias es AC, concluimos que AC > 3. Análoga-
mente, analizando los puntos C, D, E, podemos llegar a que CE > 3. Con esta información,
tenemos que AC > 3 y CE > 3, pero escogiendo los puntos A, C, E no es posible encontrar
dos de ellos cuya distancia es menor que 3, esta contradicción indica que iván no puede es
coger más de 4 puntos.

Problema 3.- En cada casilla de un tablero de 4 × 4 se escribe uno de los números 1, 2,


3 ó 4, de tal modo que no haya dos números iguales en la misma fila o en la misma
columna. Decimos que un subtablero de 2 × 2 es bacán si contiene a todos los números
del 1 al 4. ¿Cuál es el mayor número de subtableros bacanes que puede tener el tablero?
(John Cuya)
Solución:
Hay 9 subtableros de 2 × 2 en total. Supongamos que los 9 subtableros sean bacanes,
entonces el subtablero de 2 × 2 de la esquina superior izquierda debe contener a los números
1, 2, 3 y 4 en algún orden. Supongamos que A, B, C y D representan a los números 1, 2, 3
y 4, en algún orden, entonces el tablero comenzarı́a del siguiente modo
A B
C D
X Y

Luego, los números C, D, X e Y pertenecen a un subtablero de 2 × 2, entonces deben ser


los números 1, 2, 3 y 4 en algún orden, por lo cual X e Y son los mismo números que A y B,
pero como no hay dos números iguales en la misma fila ni en la misma columna, entonces A
= Y y B = X. Entonces el tablero quedarı́a del siguiente modo
A B
C D
B A
M N
Análogamente N = C y M = D. Luego
A B
C D
B A P
D C Q
Se tiene que cumplir que Q = B y P = D, de donde resulta que el subtablero central de
2 × 2 no puede contener a los números 1, 2, 3 y 4 pues contiene dos números iguales, y esto
es una contradición.
Entonces los 9 subtableros no pueden ser todos bacanes al mismo tiempo, por lo tanto el
máximo número de subtableros bacanes que se puede tener es 8, donde un ejemplo que lo
corrobora es
Comisión de Olimpiadas de la Sociedad Matemática Peruana 7

1 2 3 4
3 4 1 2
2 1 4 3
4 3 2 1
donde todos los subtableros de 2 × 2 son bacanes, excepto el central.

Problema 4.- Sean α < β < θ las raı́ces reales de la ecuación 3x3 − 3x + 1 = 0.
Si
α β θ
M= + +
β θ α
β θ α
N= + +
α β θ
Halla M + N , M N y M − N .

(Israel Dı́az )

Solución:
Comencemos observando que por el Teorema de Cardano-Viette tenemos que
0
α+β+θ =− (1)
3
−3
αβ + βθ + θα = = −1 (2)
3
1
αβθ = − (3)
3
Además, como α 6= 0 es raı́z de la ecuación inicial, podemos plantear:
1
α3 = α − (4)
3
1 1
= 3 − 3α (5)
α2 α
1 1
= 3 −3 (6)
α3 α2
y tendremos ecuaciones similares para las otras dos raı́ces β y θ.
a) Por (1) tenemos:
β+θ θ+α α+β −α −β −θ
M +N = + + = + + = −3
α β θ α β θ

b) Expandimos el producto M N :
α2 β2 θ2
  
α β θ β θ α βθ αθ αβ
MN = + + + + =3+ + + + 2+ 2 + 2
β θ α α β θ βθ αθ αβ α β θ
Comisión de Olimpiadas de la Sociedad Matemática Peruana 8

y usando (3):
 
3 3 1 1
3
 1 1
MN = 3 − 3 α + β + θ − + +
3 α3 β 3 θ 3
   
1 1 1 1 1
=3−3 α+β+θ−3· − ·3 + + −3 (por (4) y (6))
3 3 α2 β 2 θ 2
 
1 1 1
= 3 − 3 · (−1) − 3 · + + − (α + β + θ) − 1 (por (1) y (5))
α β θ
 
αβ + βθ + θα
=6−3· − 1 = 6 − 3 · (3 − 1) = 0 (por (2) y (3))
αβθ

c) Entonces, como M + N = −3 y M N = 0, tenemos {M, N } = {0, −3} y en consecuencia


|M − N | = 3. Tenemos:
β−θ θ−α α−β β−θ θ−β β−α α−β
N −M = + + = + + +
α β θ α β β θ
       
1 1 1 1 β−α θ−β
= (β − θ) − + (β − α) − = (β − θ) + (β − α)
α β β θ αβ βθ
 
1 1 (α − β)(β − θ)(θ − α)
= (β − θ)(β − α) − =−
αβ βθ αβθ

Como α < β < θ y αβθ = − 31 < 0, concluimos que N − M > 0, y en consecuencia


M − N = −3.
Comisión de Olimpiadas de la Sociedad Matemática Peruana 9

3. Nivel 3
Problema 1.- Alrededor de una mesa redonda se sientan 2n peruanos, 2n bolivianos y 2n
ecuatorianos. Si se pide que se pongan de pie todos los que tienen como vecinos, a su
derecha y a su izquierda, a personas de igual nacionalidad, ¿cuál es el mayor número
de personas que se pueden poner de pie?

Aclaración: Para que una persona se ponga de pie, sus vecinos deben tener igual na-
cionalidad entre sı́, no deben ser necesariamente de la misma nacionalidad de la persona
que se pone de pie.

(Jorge Tipe)

Solución:
Asignemos a las personas los números 1, 2, 3, . . . , 6n, en ese orden.
Analicemos a las personas que tienen asignado un número par. Si todas estas personas
se ponen de pie, entonces todas las personas que tienen asignado un número impar serı́an
de la misma nacionalidad, es decir, habrı́a 3n personas de la misma nacionalidad, esto no
es posible, pues hay 2n personas de cada nacionalidad. Supongamos ahora que todas las
personas que tienen asignado un número impar se pusieron de pie, a excepción de una de
ellas. Sin pérdida de generalidad, podemos suponer que 3, 5, 7, . . . , 6n − 1 se pusieron de
pie, pero 1 no. Luego, las personas 2, 4, 6, 8, . . . , 6n son de la misma nacionalidad, esto no
es posible, pues las personas que están junto a 1 no pueden tener la misma nacionalidad.
Concluimos que de las personas que tienen asignado un número impar, como máximo 3n − 2
se pusieron de pie.
Análogamente, de las personas que tienen asignado un número impar, como máximo 3n−2
se pusieron de pie. En total, tendrı́amos como máximo 6n − 4 personas que se pusieron de
pie.
Veamos ahora, con un ejemplo, que es posible conseguir esta cantidad (P, B, E denotan
a un peruano, boliviano y peruano, respectivamente) :

B
E B

B E

E
B

B
E

P
P
P
P P
Comisión de Olimpiadas de la Sociedad Matemática Peruana 10

en este ejemplo, se pusieron de pie todas las personas a excepción de las cuatro que están
marcadas con un cı́rculo; en total 6n − 4 personas de pie.
Problema 2.- Sean a y b números reales para los cuales se cumple:
a csc x + b cot x ≥ 1 para todos los ángulos x tales que 0 < x < 180, x en grados
sexagesimales.
Halla el mı́nimo valor de a2 + b.

(Israel Dı́az)

Solución:
Como sen x > 0, podemos multiplicara la condición inicial por sen x, y el sentido de la
desigualdad de mantiene:
a + bcos x ≥ sen x (1)
Ahora haremos lo siguiente

a ≥ sen x − bcos x = b2 + 1sen (x − θ)
en donde θ es el ángulo tal que 0o < θ < 180o , tg θ = b. Evaluando la expresión en x = 90o +θ
o x = −90o + θ (dependiendo cuál de estos ángulos está entre 0o y 180o ) tenemos que,
necesariamente √
a ≥ b2 + 1
Luego  2
2 2 1 3 3
M =a +b≥b +b+1= b+ + ≥
2 4 4
Ahora para probar que M = 34 es el mı́nimo valor de la expresión tendremos que encontrar
un par de valores

a y b tales que M = a2 + b = 34 . Para ello consideramos los valores
b = − 21 y a = 25 . Faltarı́a probar que estos valores cumplen la condición (1), que a su vez
es equivalente a la inicial, es decir,

5 1
− cos x ≥ sen x para todos los ángulos 0o < x < 180o
2 2
como ya vimos esta expresión es equivalente a la siguiente
1 ≥ sen (x − θ),
donde θ es el ángulo tal que tg θ = − 21 , la última desigualdad evidentemente es verdadera.

Problema 3.- ABC es un triángulo acutángulo con ∠ACB = 45◦ . Sean D y E puntos de
los lados BC y AC, respectivamente, tales que AB = AD = BE. Sean M , N y X los
puntos medios de BD, AE y AB, respectivamente. Si las rectas AM y BN se cortan
en el punto P , demuestra que las rectas XP y DE son perpendiculares.
Comisión de Olimpiadas de la Sociedad Matemática Peruana 11

(John Cuya)

Solución:
Observamos que el triángulo ABE es isósceles, dado que AB = BE; por lo cual si N es
punto medio de AE, entonces BN es una altura del triángulo ABE. Análogamente AM es
una altura del triángulo BAD.
Ahora en el triángulo AM C, notamos que ∠CAM = 45o . Como P pertenece a la mediatrı́z
de AE en el triángulo ABE, entonces AP = P E. Por lo tanto, tenemos que ∠AEP = 45o , y
en consecuencia EP y CB son paralelos. Análogamente DP y CA son paralelos; y además
CEP D es un paralelogramo.
B

P D

A C
N E

Por otro lado, como CD = EP = AP , DP = BP y ∠AP B = ∠CDP = 135o , con-


cluimos que los triángulos AP B y CDP son congruentes. El segmento DE es mediana del
triángulo CDP , y P X es mediana del triángulos AP B, como los lados correspondientes de
los triángulos AP B y CDP son perpepndiculares, por la congruencia, concluimos que sus
medianas también lo son, es decir, P X y DE son perpendiculares.

Problema 4.- Se pintan todos los puntos del plano que tienen ambas coordenadas enteras,
usando los colores rojo, verde y amarillo. Si los puntos se pintan de modo que haya por
lo menos un punto de cada color, demuestra que siempre existen tres puntos X, Y y Z,
de colores distintos, tales que ∠XY Z = 45◦ .

(Jorge Tipe )

Solución:
Llamaremos puntos enteros a los puntos con ambas coordenadas enteras y diagonales a
las rectas de pendiente ±1. Comenzaremos con el siguiente:
Lema.- Existen dos puntos enteros consecutivos sobre una diagonal que tienen colores
distintos.
Prueba.- Supongamos lo contrario, entonces todos los puntos enteros (x, y) con x + y par
deben tener el mismo color, y todos los puntos enteros (x, y) con x + y impar deben tener
Create PDF with GO2PDF for free, if you wish to remove this line, click here to buy Virtual PDF Printer
Comisión de Olimpiadas de la Sociedad Matemática Peruana 12

el mismo color también. Por lo tanto, todos los puntos enteros del plano han sido pintados
usando como máximo 2 colores, esto es una contradicción.
Supongamos sin pérdida de generalidad, que los dos puntos del Lema son: R de color rojo,
y V de color verde, y que están dispuestos de la forma:

V
R

Vamos a dividir el problema en dos casos.


Caso 1: existe un punto amarillo A sobre la recta RV . Supongamos sin pérdida
de generalidad que el punto V está entre A y R.

V
R
P

Sea P el punto que está en la misma horizontal que R y misma vertical que A. Si P es
verde tomamos ∠ARP = 45◦ , si P es rojo tomamos ∠V AP = 45◦ , y si P es amarillo
tomamos ∠V RP = 45◦ .
Caso 2: la recta RV sólo contiene puntos rojos y verdes. Tomamos un punto
Create PDF with GO2PDF for free, if you wish to remove this line, click here to buy Virtual PDF Printer
amarillo A y trazamos por A una horizontal y una vertical, que cortan la recta RV en
P y Q, respectivamente.

P
A
Create PDF with GO2PDF for free, if you wish to remove this line, click here
Si P y Q son de colores distintos, tomamos ∠AP Q = 45◦ . to buy Virtual PDF Printer
Si P y Q son de colores iguales, digamos que ambos son de color rojo, por el Lema
tenı́amos un punto verde V sobre P Q. Si V está debajo de Q tomamos ∠AQV = 45◦ ,
si V está encima de Q tomamos ∠AP V = 45◦ .

http://www.onemperu.wordpress.com

También podría gustarte